Analysis-Kurvendiskussion-Gebrochenrationale Funktion

1 2 3 4 5 6 7 8 9 10 11 12 13 14 15 16 17 18 19 20 21 22 23 24 25 26 27 28 29 30 31 32 33 34 35 36 37 38 39 40 41 42 43 44 45 46 47 48 49 50 51 52 53 54 55 56 57 58 59 60 61 62 63
Beispiel Nr: 58
$\begin{array}{l} \text{Gesucht:}\\ \text{Definitions- und Wertebereich} \\ \text{Grenzwerte} \\ \text{Symmetrie} \\ \text{Nullstellen - Schnittpunkt mit der x-Achse} \\ \text{Ableitungen - Stammfunktion} \\ \text{Extremwerte - Monotonie} \\ \text{Wendepunkte - Krümmung} \\ \text{Funktion:}f\left(x\right)=\displaystyle \frac{ x^2+3x+3}{ 3x+6} \ <br/> \bullet \text{Funktion/Faktorisieren} \\ f\left(x\right)=\displaystyle \frac{ x^2+3x+3}{ 3x+6} \\ \text{Zaehler faktorisieren: } \\ x^2+3x+3 = 0 \\ \\ 1x^{2}+3x+3 =0\\ x_{1/2}=\displaystyle\frac{-3 \pm\sqrt{3^{2}-4 \cdot 1 \cdot 3}}{2\cdot1}\\ x_{1/2}=\displaystyle \frac{-3 \pm\sqrt{-3}}{2}\\ \text{Diskriminante negativ keine Lösung} \\ \\\text{Nenner faktorisieren:} \\ 3x+6 = 0 \\ \\ 3 x+6 =0 \qquad /-6 \\ 3 x= -6 \qquad /:3 \\ x=\displaystyle\frac{-6}{3}\\ x=-2 \\ \underline{x_1=-2; \quad1\text{-fache Nullstelle}} \\ \\ \text{Faktorisierter Term:}\\ f\left(x\right)=\displaystyle\frac{(x^2+3x+3)}{3(x+2)} \\ \\ \bullet\text{Definitionsbereich:}\qquad \mathbb{D} = \mathbb{R}\setminus \left\{-2\right\} \\ f\left(x\right)= \displaystyle \frac{ \frac{1}{3}x^2+x+1}{ x+2} \\ Polynomdivision:\\\small \begin{matrix} ( \frac{1}{3}x^2&+x&+1&):( x +2 )= \frac{1}{3}x +\frac{1}{3} \\ \,-( \frac{1}{3}x^2&+\frac{2}{3}x) \\ \hline & \frac{1}{3}x&+1&\\ &-( \frac{1}{3}x&+\frac{2}{3}) \\ \hline && \frac{1}{3}&\\ \end{matrix} \\ \normalsize \\ \\ f(x)= \frac{1}{3}x+\frac{1}{3}+\frac{ \frac{1}{3}}{ x+2} \\ \\ \bullet \text{1. Ableitungen und 2.Ableitung} \\f'\left(x\right)=\frac{( \frac{2}{3}x+1)\cdot( x+2)-( \frac{1}{3}x^2+x+1)\cdot 1}{( x+2)^2}\\ = \frac{( \frac{2}{3}x^2+2\frac{1}{3}x+2)-( \frac{1}{3}x^2+x+1)}{( x+2)^2}\\ = \frac{ \frac{1}{3}x^2+1\frac{1}{3}x+1}{( x+2)^2}\\ = \frac{ \frac{1}{3}x^2+1\frac{1}{3}x+1}{( x+2)^2}\\ f''\left(x\right)=\frac{( \frac{2}{3}x+1\frac{1}{3})\cdot( x^2+4x+4)-( \frac{1}{3}x^2+1\frac{1}{3}x+1)\cdot( 2x+4)}{( x^2+4x+4)^2}\\ = \frac{( \frac{2}{3}x^3+4x^2+8x+5\frac{1}{3})-( \frac{2}{3}x^3+4x^2+7\frac{1}{3}x+4)}{( x^2+4x+4)^2}\\ = \frac{4,44\cdot 10^{-16}x^2+\frac{2}{3}x+1\frac{1}{3}}{( x^2+4x+4)^2}\\ = \frac{4,44\cdot 10^{-16}x^2+\frac{2}{3}x+1\frac{1}{3}}{( x^2+4x+4)^2} \\ \\ \bullet \text{Nullstellen / Schnittpunkt mit der x-Achse:} \\Zaehler =0 \\ \frac{1}{3}x^2+x+1 = 0 \\ \\ \\ \bullet \text{Vorzeichentabelle:} \\ \begin{array}{|c|c|c|c||} \hline & x < &-2&< x\\ \hline f(x)&-&0&+\\ \hline \end{array}\\ \\ \underline{\quad x \in ]-2;\infty[\quad f(x)>0 \quad \text{oberhalb der x-Achse}}\\ \\ \underline{\quad x \in ]-\infty;-2[\quad f(x)<0 \quad \text{unterhalb der x-Achse}} \\ \\ \bullet\text{Grenzwerte und Asymtoten: } \\ \\ \lim\limits_{x \rightarrow \infty}{\displaystyle \frac{x^2( 1+\dfrac{3}{x}+\dfrac{3}{x^2}) }{x( 3+\dfrac{6}{x}) }}=\infty \\\lim\limits_{x \rightarrow -\infty}{\displaystyle \frac{x^2( 1+\dfrac{3}{x}+\dfrac{3}{x^2}) }{x( 3+\dfrac{6}{x}) }}=\infty \\ \\ \text{Schiefe Asymptote:} y= \frac{1}{3}x+\frac{1}{3} \\\lim\limits_{x \rightarrow -2^+}{\displaystyle\frac{\frac{1}{3}(x^2+3x+3)}{(x+2)}}=\infty\\ \lim\limits_{x \rightarrow -2^-}{\displaystyle\frac{\frac{1}{3}(x^2+3x+3)}{(x+2)}}=-\infty\\ \\ \text{Vertikale Asymptote (Polstelle): } x=-2\\ \\ \\ \bullet \text{Extremwerte/Hochpunkte/Tiefpunkte:} \\f'(x)=\displaystyle \frac{ \frac{1}{3}x^2+1\frac{1}{3}x+1}{ x^2+4x+4} = 0 \\ \\ \\ \frac{1}{3}x^{2}+1\frac{1}{3}x+1 =0 \\ x_{1/2}=\displaystyle\frac{-1\frac{1}{3} \pm\sqrt{\left(1\frac{1}{3}\right)^{2}-4\cdot \frac{1}{3} \cdot 1}}{2\cdot\frac{1}{3}} \\ x_{1/2}=\displaystyle \frac{-1\frac{1}{3} \pm\sqrt{\frac{4}{9}}}{\frac{2}{3}} \\ x_{1/2}=\displaystyle \frac{-1\frac{1}{3} \pm\frac{2}{3}}{\frac{2}{3}} \\ x_{1}=\displaystyle \frac{-1\frac{1}{3} +\frac{2}{3}}{\frac{2}{3}} \qquad x_{2}=\displaystyle \frac{-1\frac{1}{3} -\frac{2}{3}}{\frac{2}{3}} \\ x_{1}=-1 \qquad x_{2}=-3 \\ \underline{x_2=-3; \quad1\text{-fache Nullstelle}} \\\underline{x_3=-1; \quad1\text{-fache Nullstelle}} \\f''(-3)=-\frac{2}{3} \\ f''(-3)<0 \Rightarrow \underline{\text{Hochpunkt:} (-3/-1)} \\ f''(-1)=\frac{2}{3}>0 \Rightarrow \underline{\text{Tiefpunkt:} (-1/\frac{1}{3})} \\ \\ \, \, \\\bullet\text{Monotonie/ streng monoton steigend (sms)/streng monoton fallend (smf) } \\f'\left(x\right)=\displaystyle \frac{ \frac{1}{3}x^2+1\frac{1}{3}x+1}{ x^2+4x+4}\\ \,\text{Zaehler} =0 \\\underline{x_4=-3; \quad1\text{-fache Nullstelle}} \\\underline{x_5=-1; \quad1\text{-fache Nullstelle}} \\\, \\\text{Nullstellen des Nenners aus f(x) übernehmen} \\\underline{x_6=-2; \quad1\text{-fache Nullstelle}} \\\, \\ \begin{array}{|c|c|c|c|c|c|c|c|c|c||} \hline & x < &-3&< x <&-2&< x <&-1&< x\\ \hline f'(x)&+&0&-&0&-&0&+\\ \hline \end{array}\\ \\ \underline{\quad x \in ]-\infty;-3[\quad \cup \quad]-1;\infty[\quad f'(x)>0 \quad \text{streng monoton steigend }}\\ \\ \underline{\quad x \in ]-3;-2[\quad \cup \quad]-2;-1[\quad f'(x)<0 \quad \text{streng monoton fallend }} \\ \\ \bullet\text{Kruemmung} \\f''\left(x\right)=\displaystyle \frac{ 4,44\cdot 10^{-16}x^2+\frac{2}{3}x+1\frac{1}{3}}{ x^4+8x^3+24x^2+32x+16}\\ \,Zaehler =0 \\\\ \\ 4,44\cdot 10^{-16}x^{2}+\frac{2}{3}x+1\frac{1}{3} =0 \\ x_{1/2}=\displaystyle\frac{-\frac{2}{3} \pm\sqrt{\left(\frac{2}{3}\right)^{2}-4\cdot 4,44\cdot 10^{-16} \cdot 1\frac{1}{3}}}{2\cdot4,44\cdot 10^{-16}} \\ x_{1/2}=\displaystyle \frac{-\frac{2}{3} \pm\sqrt{\frac{4}{9}}}{8,88\cdot 10^{-16}} \\ x_{1/2}=\displaystyle \frac{-\frac{2}{3} \pm\frac{2}{3}}{8,88\cdot 10^{-16}} \\ x_{1}=\displaystyle \frac{-\frac{2}{3} +\frac{2}{3}}{8,88\cdot 10^{-16}} \qquad x_{2}=\displaystyle \frac{-\frac{2}{3} -\frac{2}{3}}{8,88\cdot 10^{-16}} \\ x_{1}=-2 \qquad x_{2}=-1,5\cdot 10^{15} \\ \underline{x_7=-1,5\cdot 10^{15}; \quad1\text{-fache Nullstelle}} \\\underline{x_8=-2; \quad1\text{-fache Nullstelle}} \\\, \\\text{Nullstelle des Nenners aus f(x) übernehmen} \\\underline{x_9=-2; \quad1\text{-fache Nullstelle}} \\ \begin{array}{|c|c|c|c|c|c|c|c|c|c||} \hline & x < &-1,5\cdot 10^{15}&< x <&-2&< x <&-2&< x\\ \hline f''(x)&-&0&-&0&-&0&+\\ \hline \end{array}\\ \\ \underline{\quad x \in ]-2;\infty[\quad f''(x)>0 \quad \text{linksgekrümmt}}\\ \\ \underline{\quad x \in ]-\infty;-1,5\cdot 10^{15}[\quad \cup \quad]-1,5\cdot 10^{15};-2[\quad \cup \quad]-2;-2[\quad f''(x)<0 \quad \text{rechtsgekrümmt}} \\ Funktionsgraph und Wertetabelle \\ \end{array}$